Problema con respecto a la flotabilidad

Una canica esférica de radio. 1 cm está metido en un agujero circular un poco más pequeño que su propio radio (a efectos de cálculo, ambos son iguales) en el fondo de un balde lleno de altura 10 cm. Encuentre la fuerza sobre la canica debido al agua.

Siempre me han preocupado problemas como este. ¿La canica no desplaza cierto volumen de fluido? ¿No debería actuar sobre él una fuerza de flotación? Sin embargo, en este problema, la respuesta es igual al producto de la presión y el área de proyección....

Y, cuando me encontré con este problema similar: -

Una bola de acero flota en un canal de mercurio. Si llenamos la parte vacía del abrevadero con agua, ¿qué le sucede a la bola de acero?

La respuesta a esta es que la bola de acero sube.

Aquí, en lugar de multiplicar la presión y el área de proyección, y argumentar que actúa una fuerza neta hacia abajo, argumentamos que la bola de acero desplaza el agua y provoca que actúe una fuerza de flotación hacia arriba.

Mi pregunta es, ¿cuándo se sabe qué fuerza aplicar?

Respuestas (2)

Todo depende del contacto fluido.

La flotabilidad se produce debido a las diferencias de presión hidrostática en un objeto sumergido o flotante.

Para objetos sumergidos o flotantes, la presión del fluido actúa sobre el volumen sumergido. Porque la presión del fluido aumenta con la profundidad debido a la hidrostática; cuando sumerges un objeto, la presión en la parte superior es menor que la presión en la parte inferior. Esto provoca la fuerza ascendente neta sobre el objeto que llamamos flotabilidad. Mientras el fluido esté debajo del objeto, tendrá fuerza de flotación.

Cuando su canica está en el fondo del balde con solo líquido encima, no es lo mismo que estar sumergida. No hay un fluido de mayor presión debajo de él, solo un fluido de alta presión encima de él, por lo que la fuerza neta debida al fluido actúa hacia abajo, no hacia arriba como lo hace cuando las caras superior e inferior del objeto tienen presión actuando sobre ellos.

TL; DR : debe ver si el fluido realmente rodea la pelota como en el segundo caso, o si solo actúa sobre la pelota, como en el primer caso.

Ty, esto ayudó. Pero en el segundo qn, dado que la densidad del acero es > la mitad de la densidad del mercurio, el mármol debe sumergirse más de la mitad, y la fuerza neta aplicada por el agua debe ser hacia abajo...
@Aspirant No, la fuerza neta sigue siendo ascendente. La presión aumenta con la profundidad, por lo que incluso cuando está completamente sumergido, hay más fuerza en el fondo que en la parte superior. Si esa fuerza es mayor que el peso, flota; y la fuerza siempre es mayor que el peso si el objeto es menos denso que el fluido.
Más información sobre eso aquí physics.stackexchange.com/questions/467610/…

Para el primer problema, debe saber que la flotabilidad surge debido a la diferencia de presión que, a su vez, surge debido a la masa de fluido por encima de cierto nivel.

Tienes razón al decir que la pelota desplazará un volumen de agua, y eso es igual a la mitad del volumen de la esfera.

Entonces podemos escribir que la fuerza que actúa debido al agua se debe a la (masa de agua sobre la pelota) g, que es solo otra forma de decir que es igual a la presión por el área de proyección,

y el volumen real de agua sobre la bola = (h*Área de la base de la región cilíndrica) - (La mitad del volumen de la esfera)

Y ahora es bastante fácil proceder.